LSAT and Law School Admissions Forum

Get expert LSAT preparation and law school admissions advice from PowerScore Test Preparation.

 donielleanita
  • Posts: 1
  • Joined: Aug 11, 2012
|
#4786
How is the answer choice C? I thought since the stimulus didn't contain strong language, the correct answer choice shouldn't either?
User avatar
 Dave Killoran
PowerScore Staff
  • PowerScore Staff
  • Posts: 5853
  • Joined: Mar 25, 2011
|
#4793
Hi Donielle,

Thanks for the question. A couple of things here that might help. First, this is an Assumption question, and you can see all sorts of levels of language regardless of what you see in the stimulus. That's actually also true for any question because of the different ways things can be phrased, although, in many First Family questions you see weaker language in the answers if the stimulus contains weak language. However, question types aside, in looking at the stimulus here, you see plenty of strong language ("cannot be...unless" and "shifts...detract from the merit").

Second, let's look more closely at the critic's argument. There are different ways to view this argument, but the first sentence is conditional, so I'll place it in those terms:

..... Premise: Novel highest quality :arrow: Most readers emotionally engaged

..... Premise: Narrative shifts :arrow: Tend to focus on author

..... Conclusion: Narrative shifts :arrow: Detract from work


Answer choice (C) is:


..... Premise: Tend to focus on author :arrow: Most readers emotionally engaged


Do you see how (C) operates as a Supporter Assumption? It is connecting pieces of the argument.

Please let me know if that helps. Thanks!
 blade21cn
  • Posts: 100
  • Joined: May 21, 2019
|
#82773
I can see the logic chain, but the premise is "such shifts tend to make most readers focus on the author," which is a "most" statement. How can the argument draw an absolute conclusion (conditional: "shifts ... detract from the merit of the work"), based on a premise that has weaker logical force?

Also, based on the logic chain the way Dave drew it, we would need another assumption that "NOT highest quality → detract from the merit of the work," or should we just equivocate those two terms? Thanks!
 Adam Tyson
PowerScore Staff
  • PowerScore Staff
  • Posts: 5153
  • Joined: Apr 14, 2011
|
#84215
You can for sure equate those two terms, blade, because if something detracts from a novel's merit then the novel cannot be of the highest quality.

Two things about your concern regarding the strength of the evidence compared to that of the conclusion:

1) It doesn't matter, for our purposes, whether the argument is good or not. Maybe a "most" premise isn't good enough to support the more absolute conclusion, maybe it is, but our only goal is to identify a necessary assumption in the argument. We aren't trying to perfect the argument, nor are we required to accept that the argument is a good one. We just have to get into the author's head and see what they were thinking!

2) A "most" statement actually CAN support a more absolute conclusion! If most of my meals are full of fat and cholesterol and lack nutritional value, then I can say those meals detract from the overall merit of my diet. If most of my bathtub is dirty, then I can say that the tub detracts from the cleanliness of the bathroom. And if most of the people in my building smoke cigarettes I can say their actions detract from the air quality in the building. A problem in some or most of a thing can absolutely detract from the whole thing!

Get the most out of your LSAT Prep Plus subscription.

Analyze and track your performance with our Testing and Analytics Package.